Brilliant Junior Integration Contest - Season 1

Hi Brilliant! Just like what Aditya Kumar and Anastasiya Romanova conducted earlier, this year we would also like to conduct an integration contest for juniors.

The aims of the Integration contest are to improve skills in the computation of integrals, to learn from each other as much as possible, and of course to have fun.

Eligibility:- People should fulfill either of the 2 following

  • 17 years or below

  • Level 4 or below in Calculus

    Eligible people here may participate in this contest.

The rules are as follows:

  • I will start by posting the first problem. If there is a user solves it, then they must post a new one.

  • You may only post a solution of the problem below the thread of problem and post your proposed problem in a new thread. Put them separately.

  • Only make substantial comment that will contribute to the discussion.

  • Make sure you know how to solve your own problem before posting it in case there is no one can answer it within 48 hours, then you must post the solution and you have a right to post another problem.

  • If the one who solves the last problem does not post his/her own problem after solving it within a day, then the one who has a right to post a problem is the last solver before him/her.

  • The scope of questions is only computation of integrals either definite or indefinite integrals.

  • It is NOT compulsory to post original problems. But make sure it has not been posted on brilliant.

  • Do not copy questions from last year's contest. If anyone found to do so he/she will be banned from taking further part in this contest

  • You are also NOT allowed to post a solution using a contour integration or residue method.

The final answer can ONLY contain the following special functions: gamma function, beta function, Riemann zeta function, digamma function,Harmonic numbers, trigonometric integral, Wallis' integral,

Please post your solution and your proposed problem in a single new thread.

Format your post is as follows:

1
2
3
4
5
6
7
**SOLUTION OF PROBLEM xxx (number of problem) :**

**[Post your solution here]**

**PROBLEM xxx (number of problem) :**

**[Post your problem here]**

The comments will be easiest to follow if you sort by "Newest":

#Calculus

Note by Rajdeep Dhingra
5 years, 2 months ago

No vote yet
1 vote

  Easy Math Editor

This discussion board is a place to discuss our Daily Challenges and the math and science related to those challenges. Explanations are more than just a solution — they should explain the steps and thinking strategies that you used to obtain the solution. Comments should further the discussion of math and science.

When posting on Brilliant:

  • Use the emojis to react to an explanation, whether you're congratulating a job well done , or just really confused .
  • Ask specific questions about the challenge or the steps in somebody's explanation. Well-posed questions can add a lot to the discussion, but posting "I don't understand!" doesn't help anyone.
  • Try to contribute something new to the discussion, whether it is an extension, generalization or other idea related to the challenge.
  • Stay on topic — we're all here to learn more about math and science, not to hear about your favorite get-rich-quick scheme or current world events.

MarkdownAppears as
*italics* or _italics_ italics
**bold** or __bold__ bold

- bulleted
- list

  • bulleted
  • list

1. numbered
2. list

  1. numbered
  2. list
Note: you must add a full line of space before and after lists for them to show up correctly
paragraph 1

paragraph 2

paragraph 1

paragraph 2

[example link](https://brilliant.org)example link
> This is a quote
This is a quote
    # I indented these lines
    # 4 spaces, and now they show
    # up as a code block.

    print "hello world"
# I indented these lines
# 4 spaces, and now they show
# up as a code block.

print "hello world"
MathAppears as
Remember to wrap math in \( ... \) or \[ ... \] to ensure proper formatting.
2 \times 3 2×3 2 \times 3
2^{34} 234 2^{34}
a_{i-1} ai1 a_{i-1}
\frac{2}{3} 23 \frac{2}{3}
\sqrt{2} 2 \sqrt{2}
\sum_{i=1}^3 i=13 \sum_{i=1}^3
\sin \theta sinθ \sin \theta
\boxed{123} 123 \boxed{123}

Comments

Problem 18:

Prove that: 01(1logx+11x12)dx1x=12+12ln(2π)12γ.\int_{0}^{1} \left(\frac{1}{\log x} + \frac{1}{1-x} -\frac{1}{2} \right) \frac{ \mathrm{d}x}{1-x} = -\frac{1}{2}+\frac{1}{2} \ln (2 \pi)-\frac{1}{2} \gamma.

The Problem has been solved by Ishan Singh

Aditya Kumar - 5 years, 2 months ago

Log in to reply

Lemma 1 : r=1nHr=(n+1)Hnn\displaystyle \sum_{r=1}^{n} H_{r} = (n+1)H_{n} - n

Proof : r=1nHr=01r=1nxr1x1dx\displaystyle \sum_{r=1}^{n} H_{r} = \int_{0}^{1} \sum_{r=1}^{n} \dfrac{x^r-1}{x-1} \mathrm{d}x

=01xn+1xnx+n(x1)2dx\displaystyle = \int_{0}^{1} \dfrac{x^{n+1} - x - nx + n}{(x-1)^2} \mathrm{d}x

=01xn+1x(x1)2dx01nx1dx\displaystyle = \int_{0}^{1} \dfrac{x^{n+1} - x}{(x-1)^2} \mathrm{d}x - \int_{0}^{1} \dfrac{n}{x-1} \mathrm{d}x

=A+B\displaystyle = \text{A} + \text{B}

Using Integration By Parts on the first integral A\text{A}, we have,

A=[xn+1xx1]01+01(n+1)xn1x1dx\displaystyle \text{A} = -\left[\dfrac{x^{n+1}-x}{x-1}\right]_{0}^{1} + \int_{0}^{1}\dfrac{(n+1)x^n - 1}{x-1} \mathrm{d}x

=n+01(n+1)xn1x1dx\displaystyle = -n + \int_{0}^{1}\dfrac{(n+1)x^n - 1}{x-1} \mathrm{d}x

    A+B=n+(n+1)01xn1x1dx\displaystyle \implies \text{A} +\text{B} = -n + (n+1) \cdot \int_{0}^{1}\dfrac{x^n - 1}{x-1} \mathrm{d}x

=(n+1)Hnn\displaystyle = (n+1)H_{n} - n

Lemma 2 : 01xn1lnxdx=ln(n+1)\displaystyle \int_{0}^{1} \dfrac{x^n - 1}{\ln x} \mathrm{d}x = \ln (n +1)

Proof : 01xn1lnx dx=010nxy dy dx\displaystyle \int_{0}^{1} \dfrac{x^n - 1}{\ln x} \ \mathrm{d}x = \int_{0}^{1} \int_{0}^{n} x^y \ \mathrm{d}y \ \mathrm{d}x

=0n01xy dx dy\displaystyle = \int_{0}^{n} \int_{0}^{1} x^y \ \mathrm{d}x \ \mathrm{d}y

=0n1y+1 dy\displaystyle = \int_{0}^{n} \dfrac{1}{y+1} \ \mathrm{d}y

=ln(n+1)\displaystyle = \ln (n+1)

Lemma 3 : 01(1ln(x)+11x)dx=γ\displaystyle \int_{0}^{1} \left(\dfrac{1}{\ln (x)} + \dfrac{1}{1-x}\right) \mathrm{d}x = \gamma

Proof : 01(11x+1ln(x))dx=limn01(1tn1ttn11lntdt)\displaystyle \int_{0}^{1} \left( \dfrac{1}{1-x} + \dfrac{1}{\ln (x)}\right) \mathrm{d}x = \lim_{n \to \infty} \int_0^1\left(\dfrac{1-t^n}{1-t}-\frac{t^{n-1}-1}{\ln t} \mathrm{d}t \right)

=limn(Hnlnn)\displaystyle = \lim _{n \to \infty} (H_{n} - \ln n)

=γ\displaystyle = \gamma

Now,

I=01(1lnx+11x12)dxx1\displaystyle \text{I} = \int_{0}^{1} \left(\dfrac{1}{\ln x} + \dfrac{1}{1-x} - \dfrac{1}{2}\right)\dfrac{\mathrm{d}x}{x-1}

=limnr=1n01(xr1lnx+xr11xxr12)\displaystyle = \lim_{n \to \infty} \sum_{r=1}^{n} \int_{0}^{1} \left(\dfrac{x^{r-1}}{\ln x} + \dfrac{x^{r-1}}{1-x} - \dfrac{x^{r-1}}{2} \right)

Using Lemma 2 and Lemma 3 , we have,

I=limnr=1n(γ+lnrHr112r)\displaystyle \text{I} = \lim_{n \to \infty} \sum_{r=1}^{n} \left(\gamma + \ln r - H_{r-1} -\dfrac{1}{2r} \right)

=limn(nγ+ln(n!)r=1nHr1Hn2)\displaystyle = \lim_{n \to \infty} \left( n\gamma + \ln (n!) - \sum_{r=1}^{n} H_{r-1} - \dfrac{H_{n}}{2} \right)

Using Lemma 1,

I=limn(nγ+ln(n!)nHn1+n1Hn2)\displaystyle \text{I} = \lim_{n \to \infty} \left( n\gamma + \ln (n!) -n H_{n-1} + n - 1 -\dfrac{H_{n}}{2} \right)

Using Stirling's Approximation, we have,

I=limn(nγ+12ln(2π)n+12ln(n)+nln(n)nHn1+n1Hn2)\displaystyle \text{I} = \lim_{n \to \infty} \left( n\gamma + \dfrac{1}{2} \ln (2\pi) - n +\dfrac{1}{2} \ln (n) + n\ln (n) -n H_{n-1} + n - 1 -\dfrac{H_{n}}{2} \right)

Simplifying using limn(Hnlnn)=γ\displaystyle \lim_{n \to \infty} (H_{n} - \ln n) = \gamma, we have,

I=12ln(2π)12γ12\displaystyle \text{I} = \boxed{\dfrac{1}{2} \ln (2\pi) -\dfrac{1}{2} \gamma - \dfrac{1}{2}}

Ishan Singh - 5 years, 2 months ago

Log in to reply

wow,that was probably why i gave up halfway through lol

i'm sorry i didn't attend the integration contest until now,there was a problem with my wifi and it got fixed

Hamza A - 5 years, 2 months ago

Next problem please.

Harsh Shrivastava - 5 years, 2 months ago

Log in to reply

@Harsh Shrivastava @Aditya Kumar May post another question if he wants.

Ishan Singh - 5 years, 2 months ago

Nice. I used Binet's formula.

Aditya Kumar - 5 years, 2 months ago

Let us start with an easy Problem.

Problem 1

Find the value of the following (x+1)2e(x+1)2dxex2dx\Large \displaystyle \dfrac{\displaystyle \int_{-\infty}^{\infty}{{(x+1)}^2 e^{-{(x+1)}^2}dx}}{\displaystyle \int_{-\infty}^{\infty}{e^{-x^2}dx}}

This problem has been solved by Aditya Kumar.

Rajdeep Dhingra - 5 years, 2 months ago

Log in to reply

Solution to Problem 1: I1=(x+1)2e(x+1)2dx{ I }_{ 1 }=\int _{ -\infty }^{ \infty }{ { \left( x+1 \right) }^{ 2 }{ e }^{ { -\left( x+1 \right) }^{ 2 } } } dx

Substitute: (x+1)2=t{ \left( x+1 \right) }^{ 2 }=t

Therefore, we get: I1=12(t)12etdt{ I }_{ 1 }=\frac { 1 }{ 2 }\int _{ -\infty }^{ \infty }{ { \left( t \right) }^{ \frac { 1 }{ 2 } }{ e }^{ { - }t } } dt

This is symmetric about x=0x=0. Therefore, we can write it as: I1=0(t)12etdt{ I }_{ 1 }=\int _{ 0 }^{ \infty }{ { \left( t \right) }^{ \frac { 1 }{ 2 } }{ e }^{ { - }t } } dt

I1=Γ(32)=12Γ(12)=π2{ I }_{ 1 }=\Gamma \left( \frac { 3 }{ 2 } \right) =\frac { 1 }{ 2 } \Gamma \left( \frac { 1 }{ 2 } \right) =\frac { \sqrt { \pi } }{ 2 }

Similarly, I2=π{ I }_{ 2 }=\sqrt { \pi }

Hence, I1I2=12\boxed{\frac{{ I }_{ 1 }}{{ I }_{ 2 }}=\frac{1}{2}}

Aditya Kumar - 5 years, 2 months ago

Problem 2:

Prove that: 01arctan(x2+2)x2+2(x2+1) dx=5π296\int _{ 0 }^{ 1 }{ \frac { \arctan { \left( \sqrt { { x }^{ 2 }+2 } \right) } }{ \sqrt { { x }^{ 2 }+2 } \left( { x }^{ 2 }+1 \right) } \ dx } =\frac { 5{ \pi }^{ 2 } }{ 96 }

This problem has been solved by Harsh Shrivastava.

Aditya Kumar - 5 years, 2 months ago

Log in to reply

Solution to Problem 2

I=01arctan(x2+2)x2+2(x2+1) dx=5π296I = \int _{ 0 }^{ 1 }{ \frac { \arctan { \left( \sqrt { { x }^{ 2 }+2 } \right) } }{ \sqrt { { x }^{ 2 }+2 } \left( { x }^{ 2 }+1 \right) } \ dx } =\frac { 5{ \pi }^{ 2 } }{ 96 }

Lets split II as A and BA \text{ and } B by using the property that arctan(x)=π/2arctan(1/x)\arctan(x) = \pi /2 -\arctan(1/x)

Thus our integral becomes I=ABI = A-B,

where A=π/201dx(1+x2)(2+x2)A = \displaystyle \pi /2\int_{0}^{1}\dfrac{dx}{(1+x^{2})(\sqrt{2+x^{2}})}

and B=01arctan(1x2+2)(1+x2)(2+x2)B= \displaystyle \int_{0}^{1}\dfrac{\arctan(\frac{1}{\sqrt{x^{2} + 2}})}{(1+x^{2})(\sqrt{2+x^{2}})}

Evaluating A,

Let's evaluate A, without the limits,

π/2dx(1+x2)(2+x2)=π/2arctanxx2+2\displaystyle \pi /2\int\dfrac{dx}{(1+x^{2})(\sqrt{2+x^{2}})} = \pi /2\arctan \dfrac{x}{\sqrt{x^{2} + 2}}

Which on substituting limits gives A = π2/12\pi ^{2}/12

Evaluating B,

Using 1/aarctan(1/a)=0adyy2+a21/a \arctan(1/a) = \int_{0}^{a} \frac{dy}{y^{2}+a^{2}},

Let a=x2+2a = \sqrt{x^2+2}.

We get B=0101dxdy(1+x2)(2+x2+y2)B = \displaystyle \int_{0}^{1}\int_{0}^{1} \dfrac{dxdy}{(1+x^{2})(2+x^{2} + y^{2})}

B=0101dxdy(1+x2)(1+y2)0101dxdy(1+x2)(2+x2+y2)B = \displaystyle \int_{0}^{1}\int_{0}^{1} \dfrac{dxdy}{(1+x^{2})(1+y^{2})} - \int_{0}^{1}\int_{0}^{1} \dfrac{dxdy}{(1+x^{2})(2+x^{2} + y^{2})}

2B=0101dxdy(1+x2)(1+y2)2B = \displaystyle \int_{0}^{1}\int_{0}^{1} \dfrac{dxdy}{(1+x^{2})(1+y^{2})}

2B=(01dx1+x2)22B = \displaystyle (\int_{0}^{1}\dfrac{dx}{1+x^{2}})^{2}

Thus B=π2/32B = \pi^{2} / 32

Hence I=AB=5π296I = A-B=\dfrac{5\pi^{2}}{96}

Harsh Shrivastava - 5 years, 2 months ago

Generalization

Ishan Singh - 5 years, 2 months ago

Problem 14:

If 0x31+eαxdx=164 \displaystyle \int_{0}^{\infty} \dfrac{x^{3}}{1+e^{\alpha x}}dx = \dfrac{1}{64}
Find α \alpha given that , α \alpha is a positive constant and ζ(4)=π490 \zeta(4) = \dfrac{\pi^{4}}{90}

Problem has been solved by Rajdeep Dhingra and Aareyan Manzoor at the same instant. Aareyan has been given credits due to his solution.

A Former Brilliant Member - 5 years, 2 months ago

Log in to reply

I=0x3n=1(eαx)ndxI=\int_0^\infty x^3 \sum_{n=1}^\infty (-e^{-\alpha x})^n dx I=n=1(1)n0x3eαnxdx=n=1(1)n6(αn)4I= \sum_{n=1}^\infty (-1)^n \int_0^\infty x^3 e^{-\alpha n x}dx=\sum_{n=1}^\infty (-1)^n \dfrac{6}{(\alpha n)^4} I=6α4(123)ζ(4)=164I=\dfrac{6}{\alpha^4} (1-2^{-3}) \zeta(4)=\dfrac{1}{64} the result follows.

Aareyan Manzoor - 5 years, 2 months ago

Is the answer α=π(5615)14\huge \displaystyle \alpha = \pi {\left(\frac{56}{15}\right)}^{\frac14} ?

Rajdeep Dhingra - 5 years, 2 months ago

Problem 6:

Find a closed form of the indefinite integral Hxdx\int H_{x} dx

Where HxH_x denotes harmonic number

This problem has been solved by Aditya Kumar.

Aareyan Manzoor - 5 years, 2 months ago

Log in to reply

Solution to Problem 6:

I'll use the definition: Hx=ψ(x+1)+γ{ H }_{ x }=\psi \left( x+1 \right) +\gamma

Hxdx=(ψ(x+1)+γ)dx\int { { H }_{ x }dx } =\int { \left( \psi \left( x+1 \right) +\gamma \right) dx }

We know that ψ(x)=d(x!)dx\displaystyle \psi \left( x \right) =\frac { d\left( x! \right) }{ dx } . Hence, using this, we get: Hxdx=ln(x!)+γx+c\boxed{\displaystyle \int { { H }_{ x }dx } =ln\left( x! \right) +\gamma x+c}

Aditya Kumar - 5 years, 2 months ago

Problem 10

Let S=sin2(x)+sin4(x)32+sin6(x)52+ Upto Infinity S = \sin^2{(x)} + \dfrac{sin^4{(x)}}{3^2} + \dfrac{sin^6{(x)}}{5^2} + \cdots \text{ Upto Infinity }

Prove that 0π/2S dx=π24π2\int_{0}^{\pi /2} {S \ dx} = \dfrac{{\pi}^2}{4} - \dfrac{\pi}2

The Problem has been solved by Ishan Singh.

Rajdeep Dhingra - 5 years, 2 months ago

Log in to reply

I have come up with several ways to prove it. Here is one of the more interesting methods.

Lemma: f(a)=11+a+r=1[1(2r+a+1)k=1r(2k12k)]=π2a+1Γ(a+1)Γ2(a+22);a>1 \displaystyle f(a) = \dfrac{1}{1+a} + \sum_{r=1}^{\infty} \left[ \dfrac{1}{(2r+a+1)}\prod_{k=1}^{r}\left(\dfrac{2k-1}{2k}\right) \right] = \dfrac{\pi}{2^{a+1}} \dfrac{\Gamma (a+1)}{\Gamma^2\left(\dfrac{a+2}{2}\right)} \quad ; \quad a > -1

Proof: I have proved it here

Now,

I=0π/2S dx=r=101sin2r(2r1)2dx\displaystyle \text{I} = \int_{0}^{\pi /2} {S \ \mathrm{d}x} = \sum_{r=1}^{\infty}\int_{0}^1 \dfrac{\sin ^{2r}}{(2r-1)^2} \mathrm{d}x

=12r=1B(r+12,12)(2r1)2\displaystyle = \dfrac{1}{2} \sum_{r=1}^{\infty} \dfrac{\operatorname{B}\left(r+\frac{1}{2} , \frac{1}{2} \right)}{(2r-1)^2}

=12r=1Γ(r+12)Γ(12)(2r1)2Γ(r+1)\displaystyle = \dfrac{1}{2} \sum_{r=1}^{\infty} \dfrac{\Gamma\left(r+\frac{1}{2} \right) \Gamma \left(\frac{1}{2} \right)}{(2r-1)^2 \Gamma(r+1)}

=π2r=1Γ(r+12)(2r1)2Γ(12)Γ(r+1)\displaystyle = \dfrac{\pi}{2} \sum_{r=1}^{\infty} \dfrac{\Gamma\left(r+\frac{1}{2} \right) }{(2r-1)^2 \Gamma \left(\frac{1}{2}\right) \Gamma(r+1)}

Changing the index of summation and using Γ(x+1)=xΓ(x)\Gamma(x+1) = x\Gamma(x), we have,

I=π4r=0Γ(r+12)(r+1)(2r+1)Γ(12)Γ(r+1)\displaystyle \text{I} = \dfrac{\pi}{4} \sum_{r=0}^{\infty} \dfrac{\Gamma\left(r+\frac{1}{2} \right)}{(r+1)(2r+1) \Gamma \left(\frac{1}{2}\right) \Gamma(r+1)}

=π2[r=0Γ(r+12)(2r+1)Γ(12)Γ(r+1)r=0Γ(r+12)(2r+2)Γ(12)Γ(r+1)]\displaystyle = \dfrac{\pi}{2} \left[\sum_{r=0}^{\infty} \dfrac{\Gamma\left(r+\frac{1}{2} \right)}{(2r+1) \Gamma \left(\frac{1}{2}\right) \Gamma(r+1)} - \sum_{r=0}^{\infty} \dfrac{\Gamma\left(r+\frac{1}{2} \right)}{(2r+2) \Gamma \left(\frac{1}{2}\right) \Gamma(r+1)}\right]

Using my solution here, we have,

I=π2[1+r=11(2r+1)k=1r(2k12k)12r=11(2r+2)k=1r(2k12k)]\displaystyle \text{I} = \dfrac{\pi}{2} \left[ 1+ \sum_{r=1}^{\infty} \dfrac{1}{(2r+1)} \prod_{k=1}^{r}\left(\dfrac{2k-1}{2k}\right) - \dfrac{1}{2} - \sum_{r=1}^{\infty} \dfrac{1}{(2r+2)} \prod_{k=1}^{r}\left(\dfrac{2k-1}{2k}\right) \right]

=π2[f(0)f(1)]\displaystyle = \dfrac{\pi}{2} \left[f(0) - f(1) \right]

=π24π2\displaystyle = \boxed{\dfrac{\pi ^2}{4} - \dfrac{\pi}{2}}

Ishan Singh - 5 years, 2 months ago

What was your intended solution?

Samuel Jones - 5 years, 2 months ago

Log in to reply

More or less the same.

Rajdeep Dhingra - 5 years, 2 months ago

@Ishan Singh @Rajdeep Dhingra Do any of you know another method to solve this question?

Samuel Jones - 5 years, 2 months ago

Log in to reply

Ishan wrote that he has come up with several ways so I guess has many. @Ishan Singh

Rajdeep Dhingra - 5 years, 2 months ago

Log in to reply

@Rajdeep Dhingra My other methods include using Dilogarithm and derivative of Beta function.

Ishan Singh - 5 years, 2 months ago

Note to the Participants:

  • Please refrain from posting solution through images. It makes this note slow to load.

  • Up-vote good solutions and problems. I have see that people are not up-voting at all. This is a wrong sign in the contest. Up-voting encourages the users to improve.

Aditya Kumar - 5 years, 2 months ago

Log in to reply

Very True.I will change the note after it crosses 50 questions.

Rajdeep Dhingra - 5 years, 2 months ago

Log in to reply

Maybe you should do it now, my browser has hanged twice loading this note!

Akshay Yadav - 5 years, 2 months ago

Change it after 20. 50 is too much.

Aditya Kumar - 5 years, 2 months ago

Problem 17

Prove that if c<1c < 1, 0π/2arcsin(ccos(x)) dx=c12+c332+c552+ (Upto Infinity)\int_{0}^{\pi/2} {\arcsin{(c \cos{(x)})} \ dx} = \frac{c}{1^2} + \frac{c^3}{3^2} + \frac{c^5}{5^2} + \cdots \text{ (Upto Infinity)}

The Problem has been solved by Aditya Kumar

Rajdeep Dhingra - 5 years, 2 months ago

Log in to reply

Solution to Problem 17:

Maclaurin series of arcsin(x)\arcsin { \left( x \right) } is: arcsin(x)=n=0Γ(n+12)π(2n+1)n!x2n+1\displaystyle \arcsin { \left( x \right) } =\sum _{ n=0 }^{ \infty }{ \frac { \Gamma \left( n+\frac { 1 }{ 2 } \right) }{ \sqrt { \pi } \left( 2n+1 \right) n! } { x }^{ 2n+1 } }

Here x=ccos(x)x=c \cos(x)

0π2arcsin(ccos(x))dx=0π2n=0Γ(n+12)π(2n+1)n!(ccos(x))2n+1dx\int _{ 0 }^{ \frac { \pi }{ 2 } }{ \arcsin { \left( c\cos { \left( x \right) } \right) } dx } =\int _{ 0 }^{ \frac { \pi }{ 2 } }{ \sum _{ n=0 }^{ \infty }{ \frac { \Gamma \left( n+\frac { 1 }{ 2 } \right) }{ \sqrt { \pi } \left( 2n+1 \right) n! } { \left( c\cos { \left( x \right) } \right) }^{ 2n+1 } } dx }

0π2arcsin(ccos(x))dx=n=0Γ(n+12)c2n+1π(2n+1)n!0π2(cos(x))2n+1dx\int _{ 0 }^{ \frac { \pi }{ 2 } }{ \arcsin { \left( c\cos { \left( x \right) } \right) } dx } =\sum _{ n=0 }^{ \infty }{ \frac { \Gamma \left( n+\frac { 1 }{ 2 } \right) { c }^{ 2n+1 } }{ \sqrt { \pi } \left( 2n+1 \right) n! } \int _{ 0 }^{ \frac { \pi }{ 2 } }{ { \left( \cos { \left( x \right) } \right) }^{ 2n+1 }dx } }

Therefore, by beta function we get:

0π2arcsin(ccos(x))dx=n=0(Γ(n+12)c2n+1π(2n+1)n!12Γ(12)Γ(n+1)Γ(n+32))\int _{ 0 }^{ \frac { \pi }{ 2 } }{ \arcsin { \left( c\cos { \left( x \right) } \right) } dx } =\sum _{ n=0 }^{ \infty }{ \left( \frac { \Gamma \left( n+\frac { 1 }{ 2 } \right) { c }^{ 2n+1 } }{ \sqrt { \pi } \left( 2n+1 \right) n! }\frac { 1 }{ 2 } \frac { \Gamma \left( \frac { 1 }{ 2 } \right) \Gamma \left( n+1 \right) }{ \Gamma \left( n+\frac { 3 }{ 2 } \right) } \right) }

Hence, on simplifying, we get: 0π/2arcsin(ccos(x)) dx=c12+c332+c552+ (Upto Infinity)\boxed{ \displaystyle \int_{0}^{\pi/2} {\arcsin{(c \cos{(x)})} \ dx} = \frac{c}{1^2} + \frac{c^3}{3^2} + \frac{c^5}{5^2} + \cdots \text{ (Upto Infinity)}}

Aditya Kumar - 5 years, 2 months ago

Problem 9

Prove 0π/2xln(tan(x))dx=7ζ(3)8\int_0^{\pi/2} x\ln(\tan(x)) dx= \dfrac{7\zeta(3)}{8}

The Problem has been solved by Rajdeep Dhingra.

Aareyan Manzoor - 5 years, 2 months ago

Log in to reply

Solution to Problem 9

Page 1 Page 1

Page 2 Page 2

Rajdeep Dhingra - 5 years, 2 months ago

Log in to reply

The second page is not at all clear. Please LaTeX your solution.

Aditya Kumar - 5 years, 2 months ago

Problem 23

Prove that for an integer n2n\ge2

01{1xn}dx=nn1ζ(n)\displaystyle\int _{ 0 }^{ 1 }{ \left\{ \frac { 1 }{ \sqrt [ n ]{ x } } \right\} } dx=\frac { n }{ n-1 } -\zeta (n)

hint:let x=1unx=\frac { 1 }{ u^{ n } }

Hamza A - 5 years, 2 months ago

Log in to reply

Let I=01{1xn}dx\displaystyle \text{I} = \displaystyle\int _{ 0 }^{ 1 }{ \left\{ \frac { 1 }{ \sqrt [ n ]{ x } } \right\} } dx

Substitute x=1tn\displaystyle x = \frac{1}{t^n}

    I=n1{t}tn+1dt\displaystyle \implies \text{I} = n \int_{1}^{\infty} \dfrac{ \left \{t \right \} }{t^{n+1}} dt

=nr=1rr+1trtn+1dt\displaystyle = n \sum_{r=1}^{\infty} \int_{r}^{r+1} \dfrac{t-r}{t^{n+1}} dt

=nr=1[t1n1n+rtnn]rr+1\displaystyle = n \sum_{r=1}^{\infty} \left[ \dfrac{t^{1-n}}{1-n} +r \dfrac{t^{-n}}{n} \right]_{r}^{r+1}

=nr=1(1n(n+1)(r+1)n1+1n(r+1)n)\displaystyle = -n \sum_{r=1}^{\infty} \left( \dfrac{1}{n(n+1) (r+1)^{n-1}} + \dfrac{1}{n(r+1)^n} \right)

=nn1ζ(n)\displaystyle = \dfrac{n}{n-1} - \zeta(n)

Samuel Jones - 5 years, 2 months ago

Does the Curly brackets mean fractional part ?

Rajdeep Dhingra - 5 years, 2 months ago

Log in to reply

It is pointless to continue the contest. It hasn't garnered attention like the previous original integration contests.

Samuel Jones - 5 years, 2 months ago

Log in to reply

@Samuel Jones No Problem. Still it is open. Please post the next question if you want or if don't want to post, then @Hummus a will post.

Rajdeep Dhingra - 5 years, 2 months ago

Problem 3:

Prove that 01(ln(1+x)ln(1x))x(1x2)dx=π22\displaystyle \int_{0}^{1} \dfrac{(\ln(1+x) - \ln(1-x))}{x(\sqrt{1-x^{2}})}dx=\frac{\pi^{2}}{2}

This problem has been solved by Rajdeep Dhingra.

Harsh Shrivastava - 5 years, 2 months ago

Log in to reply

Solution to Problem 3

Let us start with the integral I(a)=01ln(1+ax)x1x2 dxI(a) = \int_{0}^{1}{ \dfrac{\ln{(1+ax)}}{x\sqrt{1-x^2}} \ dx}. Differentiate with respect to a. I(a)=011(1+ax)1x2 dxI'(a) = \int_{0}^{1}{ \dfrac1{(1+ax)\sqrt{1-x^2}} \ dx} This can easily be solved by substituting 1+ax=1t1 + ax = \dfrac1t.We now get I(a)=arccos(a)1a2I'(a) = \dfrac{\arccos{(a)}}{\sqrt{1-a^2}} Now we find I(a). I(a)=arccos(a)1a2 da=12arccos2(x)+CI(a) = \int {\dfrac{\arccos{(a)}}{\sqrt{1-a^2}} \ da} = -\dfrac12\arccos^2{(x)} + C Now we put a = 1. We get I(1)=CI(1) = C Now we put a = -1. We get I(1)=π22+CI(-1) = -\dfrac{{\pi}^2}2 + C We get I(1)I(1)=π22I(1) - I(-1) = \dfrac{{\pi}^2}2

Q.E.D

Rajdeep Dhingra - 5 years, 2 months ago

Problem 8

Prove that 02πecos(θ)cos(sin(θ)) dθ=2π \int_{0}^{2 \pi} { e^{\cos{(\theta)}} \cos{(\sin{(\theta)})} \ d{\theta}} = 2\pi

Not Original

The Problem has been solved by Aareyan Manzoor.

Rajdeep Dhingra - 5 years, 2 months ago

Log in to reply

solution to problem 8

Considering the function f(t)=02πetcos(θ)cos(tsin(θ))dθf(t) = \int_{0}^{2\pi} e^{t \cos(\theta)} \cos(t\sin(\theta)) \, d\theta, we see that tf(t)=t02πetcosθ[cosθcos(tsinθ)sinθsin(tsinθ)]dθ=02πθ[etcosθsin(tsinθ)]dθ=[etcosθsin(tsinθ)]02π  =  0f(t)=0f(t)=C \begin{array}{rcl} tf'(t) & = & \displaystyle t \int_0^{2\pi} e^{t\cos\theta}\big[\cos\theta \cos(t\sin\theta) - \sin\theta\sin(t\sin\theta)\big]\,d\theta \\ & = & \displaystyle \int_0^{2\pi} \frac{\partial}{\partial \theta}\big[e^{t\cos\theta} \sin(t\sin\theta)\big]\,d\theta \\ & = & \Big[ e^{t\cos\theta}\sin(t\sin\theta)\Big]_0^{2\pi} \; = \; 0\\ &&f'(t)=0\to f(t)=C \end{array} Since f(0)=2πf(0) = 2\pi, the desired integral is f(1)=2πf(1) = \boxed{2\pi}.

Aareyan Manzoor - 5 years, 2 months ago

Problem 11

Prove that 0π/2(xsin(x))2 dx=πlog(2)\int_{0}^{\pi/2} { {\left( \dfrac{x}{\sin{(x)}} \right)}^2 \ dx} = \pi \log{(2)}

The Problem has been solved Aareyan Manzoor.

Rajdeep Dhingra - 5 years, 2 months ago

Log in to reply

solution to problem 11 uv=uvuv\int uv'=uv-\int u'v I=0π/2x2csc2(x)dxI=\int_0^{\pi/2} x^2 \csc^2(x) dx u=x2u=2x,v=csc2xv=cot(x)u=x^2\to u'=2x, v'=\csc^2 x\to v=-cot(x) I=x2cot(x)0π/2+20π/2xcot(x)dx=20π/2xcot(x)dxI=-x^2\cot(x)\mid_0^{\pi/2} +2\int_0^{\pi/2} x \cot(x) dx=2\int_0^{\pi/2} x \cot(x) dx u=xu=1,v=cot(x)v=ln(sin(x))u=x\to u'=1, v'=\cot(x)\to v=-\ln(sin(x)) I=2xln(sin(x))0π/2+20π/2ln(sin(x))dx=0π/2ln(sin2(x))dxI=2x\ln(\sin(x))\mid_0^{\pi/2}+2\int_0^{\pi/2} \ln(\sin(x)) dx=\int_0^{\pi/2} \ln(\sin^2(x)) dx B(a,1/2)=0π/22sin2a1(x)dxB(a,1/2)=\int_0^{\pi/2} 2\sin^{2a-1}(x) dx B(a,1/2)=0π/22ln(sin2(x))sin2a1(x)dxB'(a,1/2)=\int_0^{\pi/2} 2\ln(\sin^2(x))\sin^{2a-1}(x) dx B(a,1/2)=B(a,1/2)(ψ(a)ψ(a+1/2))B'(a,1/2)=B(a,1/2)(\psi(a)-\psi(a+1/2)) a=1/2a=1/2 0π/22ln(sin2(x))dx=B(1/2,1/2)(ψ(1/2)ψ(1))=π(γ(γ2ln(2)))=2πln(2)\int_0^{\pi/2} 2\ln(\sin^2(x))dx=B(1/2,1/2)(\psi(1/2)-\psi(1))=\pi(-\gamma-(-\gamma-2\ln(2)))=2\pi\ln(2) I=0π/2ln(sin2(x))dx=πln(2)I=\int_0^{\pi/2} \ln(\sin^2(x)) dx=\boxed{\pi\ln(2)}

Aareyan Manzoor - 5 years, 2 months ago

Problem 21:

Prove That:

011xxdx=n=11nn \int_{0}^1\dfrac {1}{x^x} dx = \sum_{n=1}^{\infty} \dfrac {1}{n^n}

Samuel Jones - 5 years, 2 months ago

Log in to reply

How can i write mathematical text?I solved this but i dont know how to write math text here. All i know is latex.

Vuk Stojiljkovic - 2 months, 2 weeks ago

Problem 22 :

Prove That

0sin(x2+12x)cos(x212x)xdx=π2\int_{0}^{\infty} \dfrac{\sin \left(\frac{x^2 + 1}{2x} \right) \cos \left(\frac{x^2 - 1}{2x} \right) }{x} \mathrm{d}x = \dfrac{\pi}{2}

This problem has been solved by Aditya Kumar.

Samuel Jones - 5 years, 2 months ago

Log in to reply

Solution to Problem 22:

Use the formula 2sin(A)cos(B)=sin(A+B)sin(AB)2\sin { \left( A \right) } \cos { \left( B \right) } =\sin { \left( A+B \right) } -\sin { \left( A-B \right) }

Therefore, we get: I=12{0sin(x)x+0sin(1x)xdx} I=\frac { 1 }{ 2 } \left\{ \int _{ 0 }^{ \infty }{ \frac { \sin { \left( x \right) } }{ x } } +\int _{ 0 }^{ \infty }{ \frac { \sin { \left( \frac { 1 }{ x } \right) } }{ x } dx } \right\}

It is easy to see that both the integrals are same. Hence, I=0sin(x)x=π2I=\int _{ 0 }^{ \infty }{ \frac { \sin { \left( x \right) } }{ x } } =\frac{\pi}{2}

Aditya Kumar - 5 years, 2 months ago

Log in to reply

@Rajdeep Dhingra post the next problem. I can't come up with one of this level.

Aditya Kumar - 5 years, 2 months ago

Log in to reply

@Aditya Kumar I don't have anymore problems. Any one can post the Next Question. @Samuel Jones@Harsh Shrivastava@Aareyan Manzoor

Rajdeep Dhingra - 5 years, 2 months ago

Problem 25

Let kk be a positive real number

then prove

0101{xky}dxdy=2k+1(k+1)2γk+1\displaystyle\int _{0}^{1}{\displaystyle\int _{ 0 }^{ 1 }{ \left\{ \frac { { x }^{ k } }{ y } \right\} dxdy } =\frac { 2k+1 }{ (k+1)^{ 2 } } -\frac { \gamma }{ k+1 }}

Hint:let t=xkyt=\frac{x^k}{y}

Hamza A - 5 years, 2 months ago

Log in to reply

Solution to Problem 25:

I=0101{xky}dxdyI=\int _{ 0 }^{ 1 }{ \int _{ 0 }^{ 1 }{ \left\{ \frac { { x }^{ k } }{ y } \right\} dxdy } }

I=01(01{xky}dy)dxI=\int _{ 0 }^{ 1 }{ \left( \int _{ 0 }^{ 1 }{ \left\{ \frac { { x }^{ k } }{ y } \right\} dy } \right) dx }

By IBP, we get: I=(xk+1k+1xk{t}t2dt)01+kk+101xkdxI={ \left( \frac { { x }^{ k+1 } }{ k+1 } \int _{ { x }^{ k } }^{ \infty }{ \frac { \left\{ t \right\} }{ { t }^{ 2 } } dt } \right) }_{ 0 }^{ 1 }+\frac { k }{ k+1 } \int _{ 0 }^{ 1 }{ { x }^{ k }dx }

Hence on simplifying, we get: 0101{xky}dxdy=2k+1(k+1)2γk+1\displaystyle\int _{0}^{1}{\displaystyle\int _{ 0 }^{ 1 }{ \left\{ \frac { { x }^{ k } }{ y } \right\} dxdy } =\frac { 2k+1 }{ (k+1)^{ 2 } } -\frac { \gamma }{ k+1 }}

Aditya Kumar - 5 years, 2 months ago

Problem 27

Evaluate

0101{1x+y}ndxdy\displaystyle\int _{0}^{1}{\int _{ 0 }^{ 1 }{ \left\{ \frac { 1 }{ x+y } \right\} ^{ n }dxdy }}

with nNn\in\mathbb{N}

Hint:let x+y=tx+y=t

Hamza A - 5 years, 2 months ago

Problem 4

Prove 0x(1+x)(2+x)(3+x) dx=π2(221+3)\int_{0}^{\infty} { \dfrac{\sqrt{x}}{(1+x)(2+x)(3+x)} \ dx} = \dfrac{\pi}2 \left(- 2\sqrt{2} - 1 + \sqrt{3}\right)

This problem has been solved by Harsh Shrivastava.

Rajdeep Dhingra - 5 years, 2 months ago

Log in to reply

Harsh Shrivastava - 5 years, 2 months ago

Problem 5:

Evaluate

01ln(1+x)x\displaystyle \int_{0}^{1} \dfrac{\ln (1+x)}{x}

This problem was first solved by Aaryen Manzoor using a disallowed method and then solved by Aditya Kumar using a legal method. Credit is still given to Aaryen.

Harsh Shrivastava - 5 years, 2 months ago

Log in to reply

solution to problem 5

replace x with -x to have 01ln(1x)xdx\int_0^{-1} \dfrac{\ln(1-x)}{x} dx n1xnn=ln(1x)\sum_{n≥1} \dfrac{x^n}{n}=\ln(1-x) dividing by x and integrating from 0 to a 0aln(1x)xdx=n1ann2\int_0^{a} \dfrac{\ln(1-x)}{x} dx=\sum_{n≥1} \dfrac{a^n}{n^2} We have 01ln(1x)xdx=n1(1)nn2=n1(1)nn2=n11n22n11(2n)2=12n11n2=π212\int_0^{-1} \dfrac{\ln(1-x)}{x} dx= \sum_{n≥1}\dfrac{(-1)^n}{n^2}=\sum_{n≥1} \dfrac{(-1)^n}{n^2}=\sum_{n≥1} \dfrac{1}{n^2}-2\sum_{n≥1} \dfrac{1}{(2n)^2}=\\\dfrac{1}{2}\sum_{n≥1} \dfrac{1}{n^2}=\boxed{\dfrac{\pi^2}{12}}

Aareyan Manzoor - 5 years, 2 months ago

Log in to reply

Yeah there is a hell easy method.

:)

Harsh Shrivastava - 5 years, 2 months ago

Log in to reply

@Harsh Shrivastava Alternate solution to Problem 5:\text{Alternate solution to Problem 5:}Using MacLaurin expansion of ln(1+x)\ln(1+x):ln(1+x)=r=1(1)r+1xrr\ln(1+x)=\sum_{r=1}^{\infty} (-1)^{r+1}\dfrac{x^r}{r}ln(1+x)x=r=1r=(1)r+1xr1r\Longrightarrow \dfrac{\ln(1+x)}{x}=\sum_{r=1}^{r=\infty}(-1)^{r+1}\dfrac{x^{r-1}}{r}I=01(r=1r=(1)r+1xr1r)dx\Longrightarrow I=\int_{0}^{1}(\sum_{r=1}^{r=\infty}(-1)^{r+1}\dfrac{x^{r-1}}{r})dxI=r=1(1)r+11r2=π212\Longrightarrow I=\sum_{r=1}^{\infty}(-1)^{r+1}\dfrac{\color{#D61F06}{1}}{r^2}=\dfrac{{\pi}^2}{12}.

Adarsh Kumar - 5 years, 2 months ago

Solution to Problem 5:

I=01ln(1+x)xdxI=\int _{ 0 }^{ 1 }{ \frac { \ln { \left( 1+x \right) } }{ x } dx }

I=01ln(x)1xdxI=\int _{ 0 }^{ 1 }{ \frac { \ln { \left( -x \right) } }{ 1-x } dx }

I=01ln(x)1+xdxI=-\int _{ 0 }^{ 1 }{ \frac { \ln { \left( x \right) } }{ 1+x } dx }

Now, I'll use power series of 1x+1\frac{1}{x+1}, I'll write the integral as:

I=n=0(1)n01xnlnxdxI=-\sum _{ n=0 }^{ \infty }{ { \left( -1 \right) }^{ n }\int _{ 0 }^{ 1 }{ { x }^{ n }\ln { x } dx } }

I=n=0(1)n+1(n+1)2I=\sum _{ n=0 }^{ \infty }{ \frac { { \left( -1 \right) }^{ n+1 } }{ { \left( n+1 \right) }^{ 2 } } }

I=ζ(2)2=π212\boxed{\therefore I=\frac { \zeta \left( 2 \right) }{ 2 } =\frac { { \pi }^{ 2 } }{ 12 } }

Aditya Kumar - 5 years, 2 months ago

Problem 20 prove 01ln(x2+x+1)xdx=2ζ(2)3\int_0^1 \dfrac{\ln(x^2+x+1)}{x} dx=\dfrac{2\zeta(2)}{3}

Aareyan Manzoor - 5 years, 2 months ago

Log in to reply

01ln(x2+x+1)xdx=01ln(1x3)xdx01ln(1x)xdx\displaystyle \int_{0}^1 \dfrac{\ln (x^2+x+1)}{x} dx = \int_{0}^1 \dfrac{\ln (1-x^3)}{x} dx - \int_{0}^1 \dfrac{\ln (1-x)}{x} dx

=n=101x3n1ndx+n=101xn1ndx\displaystyle = -\sum_{n=1}^{\infty} \int_{0}^1\dfrac {x^{3n-1}}{n} dx + \sum_{n=1}^{\infty} \int_{0}^1 \dfrac {x^{n-1}}{n} dx

=23ζ(2)\displaystyle = \dfrac {2}{3} \zeta (2)

Samuel Jones - 5 years, 2 months ago

Log in to reply

Awesome!

Post the next problem.

Harsh Shrivastava - 5 years, 2 months ago

well done, you are correct.

Aareyan Manzoor - 5 years, 2 months ago

Problem 24:

Prove that

ex2dx=π\int_{-\infty}^{\infty} e^{-x^2} dx = \sqrt{\pi}

The Problem has been solved by Rajdeep Dhingra.

Samuel Jones - 5 years, 2 months ago

Log in to reply

Solution to Problem 24 :

Let I=ex2 dxI = \int_{-\infty}^{\infty} { e^{- x^2} \ dx} We can write it as I=I2I = \sqrt{I^2} I=ey2 dyex2 dx I = \sqrt{\int_{-\infty}^{\infty} { e^{- y^2} \ dy} \int_{-\infty}^{\infty} { e^{- x^2} \ dx}} I=ex2+y2 dy dxI = \sqrt{\int_{-\infty}^{\infty} \int_{-\infty}^{\infty} { e^{- x^2 + y^2} \ dy \ dx }} Switching to Polar Coordinates of one integral. I=02π0er2r dr dθI = \sqrt{\int_{0}^{2\pi} { \int_{0}^{\infty} { e^{-r^2} r \ dr \ d{\theta}}}} The 'dr' one integral can be calculated by integration by parts. The other one is very easy. I=πI = \sqrt{ \pi}

Rajdeep Dhingra - 5 years, 2 months ago

Log in to reply

Nice! Please post the next problem.

Samuel Jones - 5 years, 2 months ago

Problem 26:

Prove that:

0cosxex2x dx=γ2\int_{0}^{\infty} \frac{\cos x - e^{-x^2}}{x} \ dx = -\frac{\gamma}{2}

Aditya Kumar - 5 years, 2 months ago

Log in to reply

Lemma : 0cosxexxdx=0\displaystyle \int_{0}^{\infty} \dfrac{\cos x - e^x}{x} \mathrm{d}x = 0

Proof : 0cosxexxdx=00(eyxcosxex(y+1))dx dy\displaystyle \int_{0}^{\infty} \dfrac{\cos x - e^x}{x} \mathrm{d}x = \int_{0}^{\infty} \int_{0}^{\infty} \left(e^{-yx}\cos x - e^{-x(y+1)} \right) \mathrm{d}x \ \mathrm{d}y

=0(yy2+11y+1)dy\displaystyle = \int_{0}^{\infty} \left( \dfrac{y}{y^2 + 1} - \dfrac{1}{y+1} \right) \mathrm{d}y

=[ln(y2+1y+1)]y=0y=0\displaystyle = \left[\ln \left( \dfrac{\sqrt{y^2+1}}{y+1} \right)\right]_{y=0}^{y \to \infty} = 0

Proposition : 011exxdx1exxdx=γ\displaystyle \int_{0}^{1} \dfrac{1-e^{-x}}{x} \mathrm{d}x - \int_{1}^{\infty}\dfrac{e^{-x}}{x} \mathrm{d}x = \gamma

Proof : 011exxdx1exxdx\displaystyle \int_{0}^{1} \dfrac{1-e^{-x}}{x} \mathrm{d}x - \int_{1}^{\infty}\dfrac{e^{-x}}{x} \mathrm{d}x

=001(1ex)eyxdx dy01ex(y+1)dx dy\displaystyle = \int_{0}^{\infty} \int_{0}^{1} (1-e^{-x})e^{-yx} \mathrm{d}x \ \mathrm{d}y - \int_{0}^{\infty} \int_{1}^{\infty} e^{-x(y+1)} \mathrm{d}x \ \mathrm{d}y

=0(1eyy+e(y+1)1y+1)dy0e(y+1)y+1dy\displaystyle = \int_{0}^{\infty} \left(\dfrac{1-e^{-y}}{y} + \dfrac{e^{-(y+1)} - 1}{y+1} \right) \mathrm{d}y - \int_{0}^{\infty} \dfrac{e^{(y+1)}}{y+1} \mathrm{d}y

=0(ey1y+1)dyy\displaystyle = -\int_{0}^{\infty} \left(e^{-y} - \dfrac{1}{y+1} \right)\dfrac{\mathrm{d}y}{y}

=0(ex1x+1)dxx\displaystyle = -\int_{0}^{\infty} \left(e^{-x} - \dfrac{1}{x+1} \right)\dfrac{\mathrm{d}x}{x}

Using the Lemma,

=0(cosx1x+1)dxx\displaystyle = - \int_{0}^{\infty} \left(\cos x - \dfrac{1}{x+1} \right)\dfrac{\mathrm{d}x}{x}

=γ\displaystyle = \gamma (Using my solution here (see Problem 39))

Now,

I=0cosxex2xdx\displaystyle \text{I} = \int_{0}^{\infty} \dfrac{\cos x - e^{-x^2}}{x} \mathrm{d}x

=0exex2xdx(Using Lemma)\displaystyle = \int_{0}^{\infty} \dfrac{e^{-x} - e^{-x^2}}{x} \mathrm{d}x \quad (\text{Using Lemma})

=01exxdx+01ex2xdx\displaystyle = - \int_{0}^{\infty} \dfrac{1-e^{-x}}{x} \mathrm{d}x + \int_{0}^{\infty} \dfrac{1-e^{-x^2}}{x} \mathrm{d}x

=011exxdx11exxdx+011ex2xdx+11ex2xdx\displaystyle = -\int_{0}^{1} \dfrac{1-e^{-x}}{x} \mathrm{d}x - \int_{1}^{\infty} \dfrac{1-e^{-x}}{x} \mathrm{d}x + \int_{0}^{1} \dfrac{1-e^{-x^2}}{x} \mathrm{d}x + \int_{1}^{\infty}\dfrac{1-e^{-x^2}}{x} \mathrm{d}x

=γ+011ex2xdx1ex2xdx(Using Proposition)\displaystyle = -\gamma + \int_{0}^{1} \dfrac{1-e^{-x^2}}{x} \mathrm{d}x - \int_{1}^{\infty} \dfrac{e^{-x^2}}{x} \mathrm{d}x \quad (\text{Using Proposition})

Substitute x2xx^2 \mapsto x

    I=γ+12(011exxdx1exxdx)\displaystyle \implies \text{I} = -\gamma + \dfrac{1}{2} \left( \int_{0}^{1} \dfrac{1-e^{-x}}{x} \mathrm{d}x - \int_{1}^{\infty} \dfrac{e^{-x}}{x} \mathrm{d}x \right)

=γ+γ2(Using Proposition) \displaystyle = -\gamma + \dfrac{\gamma}{2} \quad (\text{Using Proposition})

=γ2 \displaystyle = \boxed{-\dfrac{\gamma}{2}}

Ishan Singh - 5 years, 2 months ago

Log in to reply

Nice one! There's a shorter method that uses rmt!

Aditya Kumar - 5 years, 2 months ago

Log in to reply

@Aditya Kumar R.M.T. on which function?

Ishan Singh - 5 years, 2 months ago

Log in to reply

@Ishan Singh RMT on both. You'll get lims0(Γ(s)cos(πs2)12Γ(s2))=γ2\lim _{ s\rightarrow 0 }{ \left( \Gamma \left( s \right) \cos \left( \frac { \pi \, s }{ 2 } \right) -\frac { 1 }{ 2 } \, \Gamma \left( \frac { s }{ 2 } \right) \right) } =\frac { -\gamma }{ 2 }

Aditya Kumar - 5 years, 2 months ago

Log in to reply

@Aditya Kumar Nice!

Ishan Singh - 5 years, 2 months ago

Post the next problem.

Aditya Kumar - 5 years, 2 months ago

Log in to reply

@Aditya Kumar I don't have one appropriate for this contest right now, someone else may post the next problem.

Ishan Singh - 5 years, 2 months ago

Problem 7:

Prove that 01logΓ(x)cos(16πx)dx=132\int_0^1 \log \Gamma(x)\cos (16\pi x)\, dx=\frac{1}{32}

This problem has been solved by Rajdeep Dhingra.

Aditya Kumar - 5 years, 2 months ago

Log in to reply

Let us start with I=01log(Γ(x))cos(16πx) dxI = \int_{0}^{1}{ \log{(\Gamma{(x)})}cos(16\pi x) \ dx} We can also write I=01log(Γ(1x))cos(16π16πx) dx=01log(Γ(1x))cos(16πx) dxI = \int_{0}^{1}{ \log{(\Gamma{(1-x)})}cos(16\pi - 16\pi x) \ dx} = \int_{0}^{1}{ \log{(\Gamma{(1-x)})}cos(16\pi x) \ dx} Adding both we get 2I=01log(Γ(1x)Γ(x))cos(16πx) dx2I = \int_{0}^{1}{ \log{(\Gamma{(1-x)}\Gamma{(x)})}cos(16\pi x) \ dx} Using Euler Reflection Formula we get 2I=01log(π)cos(16πx) dx01log(sin(πx))cos(16πx) dx2I = \int_{0}^{1}{ \log{(\pi)}cos(16\pi x) \ dx - \int_{0}^{1}{ \log{(\sin{(\pi x)})}cos(16\pi x) \ dx}} Both are standard computations. 2I=0(116)2I = 0 - \left(-\dfrac1{16}\right) I=132\boxed{I = \frac{1}{32}} Q.E.D

Rajdeep Dhingra - 5 years, 2 months ago

Problem 12

01ln(x)ln(1x)x(1x)dx\int_0^1 \ln(x)\ln(1-x)x(1-x) dx

Problem has been solved by Harsh Shrivastava.

Aareyan Manzoor - 5 years, 2 months ago

Log in to reply

Simple derivative of beta function.

B(a,b)=01xm1(1x)n1dxB(a,b) = \displaystyle \int_{0}^{1} x^{m-1} (1-x)^{n-1}dx

Required integral is B(m,n)mn\dfrac{\partial B(m,n)}{\partial m \partial n} at m=n=2m=n=2

This evaluates toΓ(m)Γ(n)Γ(m+n)(((ψ(m)ψ(m+n))(ψ(n)ψ(m+n))ψ(m+n))\displaystyle \frac { \Gamma (m)\Gamma (n) }{ \Gamma (m+n) } (((\psi (m)-\psi (m+n))(\psi (n)-\psi (m+n))-\psi '(m+n)).

On putting the values of m and n, the final answer evaluates to 0.06840.0684.

Harsh Shrivastava - 5 years, 2 months ago

Log in to reply

This is not a complete answer. Please show how you got 0.0684.

Aditya Kumar - 5 years, 2 months ago

Problem 13:

Evaluate 0π/2ln(cosx)dx \displaystyle \int_{0}^{\pi /2} \ln (\cos x) dx

The Problem has been solved by Rajdeep Dhingra and Vighnesh Shenoy. Vighnesh solved it late by 1second. Still credit is given to Vighnesh for the next Question.

Harsh Shrivastava - 5 years, 2 months ago

Log in to reply

Solution to Problem 13

Let us start with I=0π/2log(cos(x)) dxI = \int_{0}^{\pi/2} { \log{(\cos{(x)})} \ dx} Using properties we know that I=0π/2log(sin(x)) dxI = \int_{0}^{\pi/2} { \log{(\sin{(x)})} \ dx} Adding both we get 2I=0π/2log(sin(x)cos(x)) dx2I = \int_{0}^{\pi/2} { \log{(\sin{(x)}\cos{(x)})} \ dx} Using little manipulation we get that 2I=0π/2log(sin(2x)) dx0π/2log(2) dx2I = \int_{0}^{\pi/2} { \log{(\sin{(2x)})} \ dx} - \int_{0}^{\pi/2} { \log{(2)} \ dx} Using lemma we get 2I=Iπ2log(2)I=π2log(2)2I = I - \dfrac{\pi}2 \log{(2)} \\ \Rightarrow \boxed{I = - \dfrac{\pi}2 \log{(2)} }


Proof of Lemma Used

I1=0π/2log(sin(2x)) dxI_1 = \int_{0}^{\pi/2} { \log{(\sin{(2x)})} \ dx} Substitute 2x = t. You will get I1=120πlog(sin(t)) dtI1=12(0π/2log(sin(t)) dt+π/2πlog(sin(x)) dx)I_1 = \frac12 \int_{0}^{\pi} { \log{(\sin{(t)})} \ dt} \\I_1 = \frac12 \left( \int_{0}^{\pi/2} { \log{(\sin{(t)})} \ dt} + \int_{\pi/2}^{\pi} { \log{(\sin{(x)})} \ dx} \right) Now let us substitute xπ2=tx - \frac{\pi}2 = t in the 2nd integral. Then using properties and manipulating. we get I1=0π/2log(sin(x)) dxI_1 = \int_{0}^{\pi/2} { \log{(\sin{(x)})} \ dx}

Rajdeep Dhingra - 5 years, 2 months ago

Delete this question! This question is on brilliant. See this.

Aditya Kumar - 5 years, 2 months ago

Problem 16:

Find the closed form of the following indefinite integral,

arctan(x)x11dx \displaystyle \int \dfrac{\arctan (x)}{x^{11}} dx

This problem has been solved by Rajdeep Dhingra.

A Former Brilliant Member - 5 years, 2 months ago

Log in to reply

Solution to Problem 16

It is a long tedious one. I will just mention some main steps.

Apply integration by parts with 1st function as arctan(x)\arctan{(x)} and 2nd one as dxx11\frac{dx}{x^{11}}.

Now you will get an integral 110x10(x2+1) dx\displaystyle \int{\frac1{10x^{10}(x^2 + 1)} \ dx} .

Apply partial fractions you will get 1x101x8+1x61x4+1x21x2+1\frac1{x^{10}} - \frac1{x^8} + \frac1{x^6} - \frac1{x^4} + \frac1{x^2} - \frac1{x^2 +1} Rest all is trivial Integration.

Rajdeep Dhingra - 5 years, 2 months ago

Is the answer k=15(1)k10(2k1)x2k1arctan(x)10arctan(x)10x10\displaystyle \large \sum_{k = 1}^{5}{\dfrac{{(-1)}^k}{10(2k-1){x}^{2k-1}}} - \frac{\arctan{(x)}}{10} - \frac{\arctan{(x)}}{10x^{10}} ?

Rajdeep Dhingra - 5 years, 2 months ago

Problem 19

Prove that 01ln2(1x)xdx=2ζ(3)\int_0^1 \dfrac{\ln^2(1-x)}{x}dx=2\zeta(3)

The Problem has been solved by Vighnesh Shenoy.

Aareyan Manzoor - 5 years, 2 months ago

Log in to reply

Lemma :

01xmlnn(x)dx=(1)nn!(m+1)n+1 \displaystyle \int_{0}^{1} x^{m}\ln^{n}(x)dx = \dfrac{(-1)^{n}n!}{(m+1)^{n+1}}

Consider the integral,
01xmdx=1m+1 \displaystyle \int_{0}^{1} x^{m}dx = \dfrac{1}{m+1}

Differentiate n times with respect to m,
01xmlnnxdx=(1)nn!(m+1)n+1 \displaystyle \int_{0}^{1} x^{m}\ln^{n}x dx = \dfrac{(-1)^{n}n!}{(m+1)^{n+1}}

I=01ln2(1x)xdx=01ln2(x)1xdx I = \displaystyle \int_{0}^{1} \dfrac{ln^{2}(1-x)}{x}dx = \int_{0}^{1} \dfrac{\ln^{2}(x)}{1-x} dx

Using the Taylor series for 11x \dfrac{1}{1-x}

I=k=001xkln2(x)dx I = \displaystyle \sum_{k=0}^{\infty} \int_{0}^{1} x^{k} \ln^{2}(x)dx .
Using the Lemma,
I=k=0(1)22!(k+1)3=k=12k3=2ζ(3) I = \displaystyle \sum_{k=0}^{\infty} \dfrac{(-1)^{2} 2!}{(k+1)^{3}} = \sum_{k=1}^{\infty} \dfrac{2}{k^{3}} = 2\zeta(3)

A Former Brilliant Member - 5 years, 2 months ago

Log in to reply

Post the next problem.

Harsh Shrivastava - 5 years, 2 months ago

hey @Rajdeep Dhingra i'm 16 yrs in age but level 5 in calculus , can i participate ?

A Former Brilliant Member - 4 years, 6 months ago

Problem 15:

Time for an indefinite integral:

sin9xcos8xdx\int \dfrac{\sin^9 x}{\cos^8 x} dx

This problem has been solved by Vighnesh Shenoy and Aditya Kumar at almost the same time. Credits has been given to Vighnesh.

Aareyan Manzoor - 5 years, 2 months ago

Log in to reply

Solution to Problem 15:

I=sin9xcos8xdxI=\int { \frac { \sin ^{ 9 }{ x } }{ \cos ^{ 8 }{ x } } dx }

Substitute: cosx=t\cos x=t

I=(t84t6+6t44t2+1)dtI=-\int { \left( { t }^{ -8 }-{ 4t }^{ -6 }+{ 6t }^{ -4 }-{ 4t }^{ -2 }+1 \right) } dt

I=cos(x)+sec7(x)74sec5(x)5+2sec3(x)4sec(x)\therefore I=-\cos { \left( x \right) } +\frac { \sec ^{ 7 }{ \left( x \right) } }{ 7 } -\frac { 4\sec ^{ 5 }{ \left( x \right) } }{ 5 } +2\sec ^{ 3 }{ \left( x \right) } -4\sec { \left( x \right) }

Aditya Kumar - 5 years, 2 months ago

cosx=t \cos x = t
sinxdx=dt \therefore -\sin x dx = dt
I=(1t2)4t8dt I = \displaystyle \int \dfrac{(1-t^{2})^{4}}{t^{8}}\cdot -dt
I=14t2+6t44t6+t8t8dt I =- \displaystyle \int \dfrac{1-4t^{2} + 6t^{4} -4t^{6} + t^{8}}{t^{8}} dt
I=(17t7+45t563t3+4t+t) I = - \left( \dfrac{-1}{7t^{7}} + \dfrac{4}{5t^{5}} - \dfrac{6}{3t^{3}} + \dfrac{4}{t} + t \right) + c I=(17cos7x+45cos5x63cos3x+4cosx+cosx)+c I = - \left( \dfrac{-1}{7\cos^{7} x} + \dfrac{4}{5\cos^{5} x} - \dfrac{6}{3\cos^{3}x}+ \dfrac{4}{\cos x} + \cos x \right) + c

A Former Brilliant Member - 5 years, 2 months ago

Log in to reply

Oh no! I was about to start typing...

Nihar Mahajan - 5 years, 2 months ago

Lol I was typing. So I didn't see. Post the next problem.

Aditya Kumar - 5 years, 2 months ago
×

Problem Loading...

Note Loading...

Set Loading...